LSAT 15 – Section 3 – Question 09

You need a full course to see this video. Enroll now and get started in less than a minute.

Target time: 1:18

This is question data from the 7Sage LSAT Scorer. You can score your LSATs, track your results, and analyze your performance with pretty charts and vital statistics - all with a Free Account ← sign up in less than 10 seconds

Question
QuickView
Type Tags Answer
Choices
Curve Question
Difficulty
Psg/Game/S
Difficulty
Explanation
PT15 S3 Q09
+LR
Flaw or descriptive weakening +Flaw
A
13%
159
B
82%
165
C
3%
153
D
2%
151
E
0%
146
143
150
158
+Medium 147.322 +SubsectionMedium

Here we have a flaw question, which we know from the question stem: “The argument is most vulnerable to which of the following criticisms?” Right away we know our correct answer has to do two things: be descriptively accurate, and describe the flaw of the stimulus. We also know what the wrong answers will do - describe reasoning flaws we’ve seen before, but don’t like up with our stimulus. Once we have a clear understanding of the questrion’s objective, we can proceed into structural analysis of the stimulus.

The speaker begins by telling us about the existence of correlation between chronic fatigue syndrome and low magnesium levels. We also learn that malabsorption of magnesium is associated with some types of fatigue. The author concludes on the basis of this information that raising the level of magnesium in the blood would serve as an effective cure for chronic fatigue syndrome sufferers.

The stimulus concludes a casual relationship on the basis of a subset of a correlation. Our argument lays out low magnesium levels in tandem with fatigue, but we do not know that magnesium will change anything for fatigue. It could be the case that there is a third underlying factor causing both fatigue and low magnesium levels in these sufferers. Or, potentially, that the relationship works in exactly the opposite direction - maybe fatigue causes low magnesium levels. In that case, increasing magnesium levels would do nothing to help our fatigue.

Knowing our conclusion assumes a causal relationship from a correlative one, we can jump into answer choice elimination.

Answer Choice (A) This answer choice is descriptively accurate, but not our ultimate flaw. The stimulus does bring up the correlation between magnesium malabsorption and some types of fatigue. But our argument does not need to establish that all low magnesium levels are the result of malabsorption.

Correct Answer Choice (B) This is exactly what we are looking for! This descriptively correct answer choice points out a completely different but valid way to interpret the relationship between these variables. It could be that rather than magnesium levels causing fatigue, it very well could be the other way around.

Answer Choice (C) This answer choice is descriptively accurate, but it is not the ultimate flaw with our stimulus. The fact that levels can fluctuate does not tell us they even fluctuate to a degree that would be significant.

Answer Choice (D) This answer is descriptively accurate but not the true issue in our argument. What the exact measurement for a normal level of magnesium is does not identify the misinterpretation of our correlation.

Answer Choice (E) While this answer choice is technically accurate, it is not the ultimate problem with our stimulus. Whether or not the author tells us the most effective way of raising magnesium levels does point out the incorrect interpretation of our correlation.

Take PrepTest

Review Results

Leave a Reply